Step 1: Understanding the fringe width formula. The fringe width in Young’s double-slit experiment is given by: \[ \beta = \frac{\lambda D}{d}, \] where: - \( \lambda \) is the wavelength of the light, - \( D \) is the distance between slits and screen, - \( d \) is the separation between the slits.
Step 2: Analyzing Assertion (A). Since \( \beta \propto \lambda \), red light (\(\lambda\) is larger) produces wider fringes than blue light (\(\lambda\) is smaller). Thus, Assertion (A) is incorrect because it states the opposite.
Step 3: Analyzing Reason (R). The fringe width is indeed proportional to the wavelength, which is a correct statement.
Since (A) is false but (R) is true, the correct choice is: \[ \boxed{\text{(2) (A) is false, but (R) is true.}} \]
A tightly wound long solenoid carries a current of 1.5 A. An electron is executing uniform circular motion inside the solenoid with a time period of 75 ns. The number of turns per meter in the solenoid is …………
In a hydraulic lift, the surface area of the input piston is 6 cm² and that of the output piston is 1500 cm². If 100 N force is applied to the input piston to raise the output piston by 20 cm, then the work done is _________ kJ.